You are on page 1of 62

PRE‐GATE‐2019

Eletronics and Communication Engineering


(Questions with Detailed Solutions)
The GA section consists of 10 questions. Questions 1 to 5 are of 1 mark each, and
Questions 6 to 10 are of 2 marks each.

Q. 1 – Q. 5 carry one mark each.


01. Find the missing number from the given alternatives

45 20 40

25 27 35 60 30 40 25 ? 35

30 30 65

(A) 36 (B) 33 (C) 45 (D) 60

01. Ans: (B)


135
Sol: As, 25 + 45 + 35 + 30 = 135 =  27
5
150
And 60 + 20 + 40 + 30 = 150 =  30
5
Similarly 25 + 40 + 35 + 65 = 165
165
  33
5
Hence option (B) is correct.

02. Identify the correct sentence as per the standard English


(A) I taught the dog to lay down and roll over.
(B) I taught the dog to lie down and roll over.
(C) I taught the dog to laid down and roll over.
(D) I taught the dog to lied down and roll over.
02. Ans: (B)
ACE Engineering Academy Hyderabad|Delhi|Bhopal|Pune|Bhubaneswar|Lucknow|Patna|Bengaluru|Chennai|Vijayawada|Vizag|Tirupati|Kukatpally|Kolkata|Ahmedabad
: 2 : PRE‐GATE_2019

03. Fill in the blank with an appropriate phrase


The gardens were ___ with lawns and flower beds.
(A) laid about (B) laid out (C) laid off (D) laid by
03. Ans: (B)

04. Out of the following four sentences, select the most suitable sentence with respect to grammar and
usage
(A) I will not leave the place until the minister does not meet me.
(B) I will not leave the place until the minister doesn’t meet me.
(C) I will not leave the place until the minister meet me.
(D) I will not leave the place until the minister meets me.
04. Ans: (D)

05. Which of the following options is the closest meaning to the underlined part of below sentence?

There was a homogeneity of outlook.

(A) diversity (B) unspoiled freshness


(C) similarity (D) stubbornness
05. Ans: (C)

Q. 6 – Q. 10 carry Two marks each.


06. Nine men and seven women can complete a piece of work in five days. The same work can be
completed by seven men and eleven women in four days. Which of the following statements is true
regarding the efficiency of the men and women?
8
(A) Men are more efficient than women by 88 %
9
8
(B) Women are more efficient than men by 88 %
9
8
(C) Men are more efficient than women by 18 %
9
8
(D) Women are more efficient than men by 18 %
9
ACE Engineering Academy Hyderabad|Delhi|Bhopal|Pune|Bhubaneswar|Lucknow|Patna|Bengaluru|Chennai|Vijayawada|Vizag|Tirupati|Kukatpally|Kolkata|Ahmedabad
: 3 : EC

06. Ans: (B)

1
Sol: 9 men and 7 women complete th of the work in 1 day
5

 45 men and 35 women can complete the work in 1 day

1
7 men and 11 women can complete th of the same work in 1 day
4

 28 men and 44 women can complete the work in 1 day

 45 men + 35 women

 28 men + 44 women

 17 men = 9 women

women 17 8
  1
men 9 9

8 8
 Women are more efficient by  100 = 88 %
9 9

Hence option (B) is correct

07. What is the approximate volume of the piece shown in the given figure which has a rectangular

prism surrounded by a triangular prism (of cross-section equilateral triangle)? The height of the

entire piece is 13 cm and that of the rectangular piece is 10 cm

10 cm
2 cm

ACE Engineering Academy Hyderabad|Delhi|Bhopal|Pune|Bhubaneswar|Lucknow|Patna|Bengaluru|Chennai|Vijayawada|Vizag|Tirupati|Kukatpally|Kolkata|Ahmedabad


: 4 : PRE‐GATE_2019

(A) 252 cm3 (B) 400 cm3 (C) 216 cm3 (D) 236 cm3

07. Ans: (A)

Sol: The total height of the piece is 13 cm and the height of the rectangular piece is 10 cm, the height of

the triangular piece = 13 – 10 = 3 cm since the triangular piece is an equilateral triangle, from its

height 3 cm, we get the side of the triangle as 2 3 cm.

3
 side   length
2
Volume of the top portion = Area × length =
4

=
4
3
 
2
 2 3  10  30 3  52cm 3

Volume of the bottom portion = 10 × 10 × 2 = 200 cm3

 Total volume = 252 cm3

 Hence option (A) is correct

ACE Engineering Academy Hyderabad|Delhi|Bhopal|Pune|Bhubaneswar|Lucknow|Patna|Bengaluru|Chennai|Vijayawada|Vizag|Tirupati|Kukatpally|Kolkata|Ahmedabad


: 5 : EC

08. In a primary school, the average weight of male students is 65.9 kg and the average weight of
female students is 57 kg. If the average weight of all the students (both male and female) is 60.3 kg
and the number of male students in the school is 66, then what is the number of female students in
the school?
(A) 152 (B) 162 (C) 168 (D) 112
08. Ans: (D)
Sol: Average weight of male students = 65.9 kg
Average weight of female students = 57.0 kg
Average weight of total students = 63.3 kg
Let the total number of students be ‘x’
65.9  66  x  66  57
Then,  60.3
x
65.9 × 66 + 57x – 57 × 66 = 60.3x
(65.9 – 57) × 66 = 3.3 x
8.9 × 66 = 3.3 x
x = 178
 Number of female students = 178 – 66 = 112
Hence option (D) is correct

09. Study the following pie charts and answer the given question
Distribution of total number of Dell laptops sold by 5 stores

T P
21% 15%
S Q
9% 25%
R
30%

Total number = 2400

ACE Engineering Academy Hyderabad|Delhi|Bhopal|Pune|Bhubaneswar|Lucknow|Patna|Bengaluru|Chennai|Vijayawada|Vizag|Tirupati|Kukatpally|Kolkata|Ahmedabad


: 6 : PRE‐GATE_2019

Distribution of number of Laptops (both Dell and Lenovo) sold by 5 stores in 2011

T P
15% 16%
S Q
11% 23%
R
35%

Total number = 4500

What is the difference between number of Laptops (both dell and Lenovo) sold by store Q and

total number of Lenovo Laptops sold by store ‘R’ and ‘S’ together?

(A) 185 (B) 99 (C) 91 (D) 119

09. Ans: (B)

23
Sol: Number of laptops (Dell and Lenovo) sold by store Q = 4500   45  23  1035
100

46
Now, total number of Laptops (Lenovo and Dell) sold by R and S together = 4500   2070
100

39
Number of Dell laptops sold by ‘R’ and ‘S’ together = 2400   936
100

Number of Lenovo laptops = 2070 – 936 = 1134

 Required difference = 1134 – 1035 = 99

ACE Engineering Academy Hyderabad|Delhi|Bhopal|Pune|Bhubaneswar|Lucknow|Patna|Bengaluru|Chennai|Vijayawada|Vizag|Tirupati|Kukatpally|Kolkata|Ahmedabad


: 7 : EC

ACE Engineering Academy Hyderabad|Delhi|Bhopal|Pune|Bhubaneswar|Lucknow|Patna|Bengaluru|Chennai|Vijayawada|Vizag|Tirupati|Kukatpally|Kolkata|Ahmedabad


: 8 : PRE‐GATE_2019

10. Stronger patent laws are needed to protect inventions from being pirated. With that protection
manufacturers would be encouraged to invest in the development of new products and
technologies. Such investment frequently results in an increase in manufacturer’s productivity.
Which of the following conclusions can most properly be drawn from the information above?
(A) Stronger patent laws tend to benefit financial institutions as well as manufacturers.
(B) Increased productivity in manufacturing is likely to be accompanied by the creation of more
manufacturing jobs.
(C) Manufacturers will decreases investment in the development of new products and
technologies unless there are stronger patent laws.
(D) Stronger patent laws would stimulate improvements in productivity for many manufacturers.
10. Ans: (D)

Sol: Stronger patent laws increase protection; protection encourages investment; investment often raises

productivity. Thus, stronger patent laws initiate a chain of events that often culminates in improved

productivity. Choice D expresses that and is, therefore, the best answer.

Choice A is inappropriate because the role, if any, that financial institutions would play in

investments is left open. The increased productivity mentioned in B may mean fewer hours of

labour for a given level of output, and may, thus, threaten jobs. Investments of the sort described in

C may already be at the lowest possible level.

Q. 11 – Q. 35 carry one mark each.

11. The root mean square value of the given current waveform is _____ (in Amperes)

i(t)
4A

2.5 3.5
3.5 1.5 0 2.5 4.5 8.5 t (sec)
2A

ACE Engineering Academy Hyderabad|Delhi|Bhopal|Pune|Bhubaneswar|Lucknow|Patna|Bengaluru|Chennai|Vijayawada|Vizag|Tirupati|Kukatpally|Kolkata|Ahmedabad


: 9 : EC

11. Ans: 3.366 (Range: 3.3 to 3.4)


Sol: The time period for a given waveform = 6sec
i( t ) 4A(1.5  t  2.5  4 sec)

  2A (2.5  t  3.5 1sec)  6 sec  T
 0 (3.5  t  4.5 1sec) 
Root mean square value

I rms 
42 4   22 1  02 1
6
64  4
  11.33  3.36 Amps
6
I rms  3.36 Amps

K
12. The forward path transfer function of a unity feedback system is G S  . If a unit ramp
SS  1
2

input is applied, the minimum possible steady state error is ___________


12. Ans: 0.5 (no range)
Sol: The steady state error is minimum to the largest value of K for which the closed loop system is
stable.
K
RH criterian : CE 1  G S  0  1  0
SS  1
2

CE  S3 + 2S2 + S + K = 0
S3 1 1
S2 2 K
S1  2  K 
   0 (S)
 2 
S0 K  0 (S)

0 < K < 2  CL stable


Largest value of K for which CL system stable is 2
2
G (S)  , H(S) 1
SS  1
2

For unit Ramp input


A
e ss  A=1
Kv
 2 
Kv = Lt S G S  Lt S  2
2 
S 0 S 0
 SS  1 
ACE Engineering Academy Hyderabad|Delhi|Bhopal|Pune|Bhubaneswar|Lucknow|Patna|Bengaluru|Chennai|Vijayawada|Vizag|Tirupati|Kukatpally|Kolkata|Ahmedabad
: 10 : PRE‐GATE_2019

13. A MOSFET carries a drain current of 1mA with VDS = 0.5 V in Saturation. Then the change in
ID(in A) is ___________, if VDS rises to 1V and  = 0.1 V1
13. Ans: 47.6 (Raneg: 45 to 49)
1 W
I
 n c ox VGS  VT  1  VDS 2 
2

1  VDS 2
Sol: D 2  2 L 
I D1 1 W
 n c ox VGS  VT 2 1  VDS1  1  VDS1
2 L
1  VDS2 1  0.11
 I D2   I D1   1mA 1.0476mA
1  VDS1 1  0.1 0.5

I D  I D 2  I D1 1.0476 mA  1mA  47.6 A

14. For the circuit shown in figure the value of R (in k) is _______ that results in VD = 0.7 V.
W 0.72 m
The MOSFET has VT = 0.5 V, n Cox = 0.4 m A/V,  and   0
L 0.18 m
+1.8V

R
VD

14. Ans: 34.375 (Range: 33 to 36)


1 W
Sol: I D   n C ox VGS  VT 
2

2 L
1 W
I D   n C ox VD  VT  2
2 L

1
I D   0.4 10 3  4 0.7  0.5
2

2
 32 A

1.8V  VD 1.8V  0.7V


R   34.375 k
ID 32 A

ACE Engineering Academy Hyderabad|Delhi|Bhopal|Pune|Bhubaneswar|Lucknow|Patna|Bengaluru|Chennai|Vijayawada|Vizag|Tirupati|Kukatpally|Kolkata|Ahmedabad


: 11 : EC

1
e ss   0.5
2

ACE Engineering Academy Hyderabad|Delhi|Bhopal|Pune|Bhubaneswar|Lucknow|Patna|Bengaluru|Chennai|Vijayawada|Vizag|Tirupati|Kukatpally|Kolkata|Ahmedabad


: 12 : PRE‐GATE_2019

15. For the regulator circuit given, load current (IL) is ________ mA. Assume that the op-amp is ideal

20V to 30V

2K
+
5V –

+
3K RL
IL

15. Ans: 2.5 (No range)


Sol: KVL
5 + 0 – IL (2K) = 0
IL = 2.5 mA

16. The number of 2 input Ex-OR gates required to construct an 8-input EX-NOR gate is ______

16. Ans: 8 (no range)

Sol:
A 1
B
5
C 2
D A = ABCDEFGH
7 8
E 1
3
F
6
G 4
H

Total number of two input Ex-OR gates are 8

ACE Engineering Academy Hyderabad|Delhi|Bhopal|Pune|Bhubaneswar|Lucknow|Patna|Bengaluru|Chennai|Vijayawada|Vizag|Tirupati|Kukatpally|Kolkata|Ahmedabad


: 13 : EC

17. In a 4-bit ripple carry adder, the full adder takes 3ns and 2ns to produce sum and carry outputs
respectively. The inputs to this adder are FH and 1H and are applied at time t = 0.At ____ nsec, the
output of the adder is 8H.
17. Ans: 7 (no range)
Sol:
1 1 1 1
00 00 00 1
0 0 0 0

At 3ns  1 1 1 02 = EH and C2 = 1 at 2ns


At 5ns  1 1 0 02 = CH and C3 = 1 at 4ns
At 7ns  1 0 0 02 = 8H and C4 = 1 at 6ns

18. In 8085 microprocessor, the Accumulator is loaded with –110 in 2's complement form. 'CY' flag is
'0' initially.
LOOP: RAL
CMC
JNC LOOP
Then LOOP executes for _______ times.
18. Ans: 9 (no range)
Sol: Initially [A] = –110 = 1111 11112 and CY = 0
(1) Loop : RAL ; [A] = 1111 1110, CY = 1
CMC ; CY = 0
JNC Loop; So Loop will be executed
(2) Loop : RAL ; [A] = 1111 1100, CY = 1
CMC ; CY = 0
JNC Loop
(3) Loop : RAL ; [A] = 1111 1000, CY = 1
CMC ; CY = 0
JNC Loop
(4) Loop : RAL ; [A] = 1111 0000 , CY = 1
CMC ; CY = 0
JNC Loop

ACE Engineering Academy Hyderabad|Delhi|Bhopal|Pune|Bhubaneswar|Lucknow|Patna|Bengaluru|Chennai|Vijayawada|Vizag|Tirupati|Kukatpally|Kolkata|Ahmedabad


: 14 : PRE‐GATE_2019

(5) Loop : RAL ; [A] = 1110 0000, CY = 1


CMC ; CY = 0
JNC Loop
(6) Loop : RAL ; [A] = 1100 0000, CY = 1
CMC ; CY = 0
JNC Loop
(7) Loop : RAL ; [A] = 1000 0000, CY = 1
CMC ; CY = 0
JNC Loop
(8) Loop : RAL ; [A] = 0000 0000, CY = 1
CMC ; CY = 0
JNC Loop ;
(9) Loop : RAL ; [A] = 0000 0000, CY = 0
CMC ; CY = 1
JNC Loop ; Loop gets terminated here
LOOP executes for 9 times.

19. A lossless transmission line having characteristic impedance of 120 is operating at

 = 5 × 108rad/s. If the velocity on the line is 2.4×108 m/s, then the shunt capacitance (in pF/m) on

the line is ______

19. Ans: 34.7 (Range 33 to 36)


Sol: Given: Z0 = 120 ,  = 5 × 108 r/s ,   2.4 108 m / s
L
For lossless line Z 0  and
C
1

LC
1
Z o 
C
1 1
C    34.7 pF
Z o 2.4 108 120

ACE Engineering Academy Hyderabad|Delhi|Bhopal|Pune|Bhubaneswar|Lucknow|Patna|Bengaluru|Chennai|Vijayawada|Vizag|Tirupati|Kukatpally|Kolkata|Ahmedabad


: 15 : EC

20. Consider a message signal m(t) having Fourier transform M(f) as shown in the figure below:

M(f)

–10000 0 10000 f (Hz)

It is known that the signal is normalized means that –1  m(t)  1. If an FM signal with

kf = 60kHz is used. The bandwidth of the modulated signal is _____ kHz.

20. Ans: 140 (no range)

f k f A m 60  10 3  1
Sol:  FM    6
fm fm 10  103

 s(t)FM is WBFM signal


As per Carson’s rule:
BW = 2( + 1) fm
 BW = 2(6+1) 10310 = 140 kHz
21. Consider the angle modulated signal s(t) = 10cos[2000t + 100t2].
The instantaneous frequency of the signal at t = 2 sec is _____ Hz.
21. Ans: 1200 (no range)
Sol: The instantaneous frequency is
1 d
fi  [2000t  100t 2 ]
2 dt
1
fi  [2000  200t ]
2
f i  1000  100 t
where t = 2
fi = 1000 + 200
fi = 1200 Hz

ACE Engineering Academy Hyderabad|Delhi|Bhopal|Pune|Bhubaneswar|Lucknow|Patna|Bengaluru|Chennai|Vijayawada|Vizag|Tirupati|Kukatpally|Kolkata|Ahmedabad


: 16 : PRE‐GATE_2019

 2  3
22. Let A    . If one of the eigen values of A is 3, then a = _____.
a 5 

22. Ans: 0.6667 (Range: 0.65 to 0.67)

Sol: Let  be the second eigen value of A.

We know that, sum of the eigen values of A = Trace of A

3+=2+5

=4

Again, product of the eigen values of A = |A|

 12 = 10 + 3a

2
a= = 0.6667
3

23. If f(x) = x2 and g(x) = x3 then, the mean value C satisfying Cauchy's Mean Value theorem in the
interval (1, 2) is ______.
23. Ans: 1.555 (Range: 1.5 to 1.6)
Sol: Here, f(x) and g(x) satisfy the conditions of Cauchy's mean value theorem.
By Cauchy's mean value theorem, there exists a value C  (1, 2) such that
f (C ) f 2  f 1
 ............ (1)
g (C ) g 2  g 1
2C 4  1
 
3C 2 8  1
14
 C= = 1.555
9

ACE Engineering Academy Hyderabad|Delhi|Bhopal|Pune|Bhubaneswar|Lucknow|Patna|Bengaluru|Chennai|Vijayawada|Vizag|Tirupati|Kukatpally|Kolkata|Ahmedabad


: 17 : EC

24. A pentagonal pyramid build up of ten wires with each 6 resistance as shown in figure is fed from
48V battery at the terminals A&C. Find the current passing through AB

A
E

B 48V
F
D

4 7
(A) 7A (B) 4A (C) A (D) A
7 4
24. Ans: (B)
A
Sol: 6
6 E
6 6
2
6 48V
B 2
2
6 F
6 6 D
6
6
C

R eq 12 || 16 A
12  16 6 I 8

28
48 B D 48V
  6 2
7
6 8
C
V 48 IT
IT    7 Amps A
R eq 48 I
7 6 8
By Current Division Rule current through AB(I) B 48V
D
7  16 6 8
I  4 Amps
16  12
C

ACE Engineering Academy Hyderabad|Delhi|Bhopal|Pune|Bhubaneswar|Lucknow|Patna|Bengaluru|Chennai|Vijayawada|Vizag|Tirupati|Kukatpally|Kolkata|Ahmedabad


: 18 : PRE‐GATE_2019

Distractor Logic:
V 48
(a) The total current I T    7A
R eq 48 / 7

(b) Option (B) is correct


(c) Option (C) are not possible
(d) Option (D) are not possible

25. The Nyquist plot over the frequency range 0 to  is shown in figure. For which the OLTF has one
pole in the RHS-plane. The range of K for which closed loop system is stable

Im

0.5K
K
1800 Re
=0 =

(A) K < 1 (B) 1 < K < 2 (C) K > 2 (D) K < 1 or K > 2
25. Ans: (C)
Sol:
0.5K

K 1+10
=0 =
For CL stability
N=+1
 0.5 K > 1 P=+1
K > 2 CL stable N=P
CL Stable

Distractor Logic:
(a) If critical point (1, j0) consider outside the loop, then option A is possible but CL system is
unstable
(b) If critical point (1, j0) consider between 0.5k to K, then option B is possible but CL
system is unstable
(c) Option (C) is correct
(d) If critical point consider in two regions then two different range of K values are possible. In
this case CL system is unstable.
ACE Engineering Academy Hyderabad|Delhi|Bhopal|Pune|Bhubaneswar|Lucknow|Patna|Bengaluru|Chennai|Vijayawada|Vizag|Tirupati|Kukatpally|Kolkata|Ahmedabad
: 19 : EC

26. Holes are injected into an n-type Ge Semiconductor, so that at sample surface P0 = 1014 cm–3.
Calculate total hole concentration at a distance of 4mm from the surface. (Given life time for holes
is 10–3 sec, diffusion constant for holes is 49cm2/sec, initial thermally generated minority
concentration is 11012 cm–3 & intrinsic concentration is 2.51013 cm–3)
(A) 1.641013 cm–3 (B) 6.41014 cm–3
(C) 1.031012 cm–3 (D) 1.741013 cm–3
26. Ans: (D)
Sol: Given data
pn(x)
pn (0)  p 0  1014 cm 3
p n x   p n o  pn 0 e
x / Lp

 p  10 sec
3

p n (0) pn 0
D P  49cm 2 / sec p n (0)
pn 0
p n 0  1  1012 cm 3
n i  2.5  1013 cm 3 0
x = 4mm = 4103m = 0.4 cm
Pn ( x )  ?
L p  D P P  49  10 3  0.221
p n ( x )  p n 0  pn (0)e  x / L P
 1  1012  1  1014 e 0.4 / 0.221
= 1.74 1013cm–3

Destructor Logic:
(a) Is wrong
If Pn 0 not considered 1.641013 cm–3 is answer

(b) Is wrong
 i2
If  n 0   6.25  1014 assumed
1  10 12

x / L
Then Pn ( x )  Pn 0  Pn (0)e p  6.4  1014 cm 3
(c) Is wrong
If Lp = Dpp assumed
 x / Lp
then Pn() = Pn 0  Pn (0)e  1.03  1012 cm 3

(d) Option (D) is correct

ACE Engineering Academy Hyderabad|Delhi|Bhopal|Pune|Bhubaneswar|Lucknow|Patna|Bengaluru|Chennai|Vijayawada|Vizag|Tirupati|Kukatpally|Kolkata|Ahmedabad


: 20 : PRE‐GATE_2019

ACE Engineering Academy Hyderabad|Delhi|Bhopal|Pune|Bhubaneswar|Lucknow|Patna|Bengaluru|Chennai|Vijayawada|Vizag|Tirupati|Kukatpally|Kolkata|Ahmedabad


: 21 : EC

27. Identify the Transfer characteristics of the circuit given if VD  0 .7 V


ON 

+
R1
D2 V0
Iin
D1

(A) (B)
R1 0.7
0.7
V0
V0 0 Iin
0 Iin
–0.7V

(C) (D) 0.7


0.7
V0
V0 0 .7
R 1
0 Iin
0 Iin

27. Ans: (D)


Sol: Iin Positive  Diode ON  V0 = VD(ON) = 0.7V
Iin Negative  Diode OFF  V0 = 0 V

Distractor Logic:

Option A: If D2 is ignored, then the transfer characteristics is shown

Option B: If R1 is ignored and diodes D2 is reversed

Option C: If D1 is neglected
Option D: Option (D) is correct

ACE Engineering Academy Hyderabad|Delhi|Bhopal|Pune|Bhubaneswar|Lucknow|Patna|Bengaluru|Chennai|Vijayawada|Vizag|Tirupati|Kukatpally|Kolkata|Ahmedabad


: 22 : PRE‐GATE_2019

28. The unity gain frequency of an Op-amp is 10kHz. Find the gain of the amplifier for a sinusoidal
input of 50kHz ?

0.5K 49.5K

+ V0
+
Vin

(A) 19.6 (B) 70.7 (C) 55.55 (D) 100


28. Ans: (A)
49.5
Sol: G  1  = 100
0.5
V0 100

Vin 1  jf
10 4
V0 100
 = 19.6
Vin 50 K  50K 
2

1  
 10K 
Distractor Logic:
Option A: Option (A) is correct
Option B: If you calculate the 3dB gain
Option C: Gain at 15kHz instead of 50kHz
Option D: DC Gain

29. If a plane electromagnetic wave traveling in free space of amplitude 50V/m strikes normally on a
perfect dielectric as shown in figure, then the value of Et (in V/m) is

 Free space
Ei Dielectric Et
Free space
r = 81, µr =1, σ = 0

2 mm
(A) 18 (B) 10 (C) 1.8 (D) 90
29. Ans: (A)
ACE Engineering Academy Hyderabad|Delhi|Bhopal|Pune|Bhubaneswar|Lucknow|Patna|Bengaluru|Chennai|Vijayawada|Vizag|Tirupati|Kukatpally|Kolkata|Ahmedabad
: 23 : EC

Sol:

Dielectric medium Free space


r2  81, Et
freespace
E i  50 V / m 120 120
2    η2 = 120πΩ
1 120  81 9
 Et .
2mm

Transmission coefficient
 120 
2 
Et 22  9 
  = 0.2
E i 2  1  120 
  120 
 9 
Et = 0.2 × 50 = 10 V/m

Et 2 2 2  120 29
   = 1.8
Et 2  2 120 1  9
120 
9
Et = 1.8 × 10
 Et = 18 V/m

Destractor Logic:
(a) Option (A) is correct
(b) If we take amplitude of transmitted wave in medium 2 only, then Et = 10 V/m. Which is
incorrect option
(c) During simplification, if we forgot to multiply with 10, then Et = 1.8 V/m.
Et
 1 .8
E t  Et = 1.8
Which is wrong option
(d) During simplification in the third medium (free space) if we take directly Ei = 50V/m then
E
Et = 90V/m i.e. t  1.8  Et=1.850 = 90 V/m
Ei
Which is incorrect answer.
ACE Engineering Academy Hyderabad|Delhi|Bhopal|Pune|Bhubaneswar|Lucknow|Patna|Bengaluru|Chennai|Vijayawada|Vizag|Tirupati|Kukatpally|Kolkata|Ahmedabad
: 24 : PRE‐GATE_2019

30. Given X(z) = 2z2 – 5z + 5z–1 – 2z–2. Then the signal x(n) is _____
(A) Non-Causal & Odd symmetric (B) Causal & Odd symmetric
(C) Non-Causal & Even symmetric (D) Causal & Even symmetric
30. Ans: (A)
 z  n0
Sol: (n  n 0 ) 
Z.T

X(z) = 2z2 – 5z + 5z–1 – 2z–2


Apply inverse z-transform
x(n) = 2(n+2) – 5(n + 1) + 5(n – 1) – 2(n – 2)
x(n) = {2, –5, 0 , 5, –2}

x(n)  0 for n < 0. So, non-causal


x(–n) = –x(n). So, odd symmetric

Distractor Logic:
Option (A): Option (A) is correct
Option (B): As no arrow mark is given misinterpretation of feeling signal starts from n = 0
Option (C): As the sample amplitudes are having alternate sign misconception of even symmetry
Option (D): As the sample amplitudes are having alternate sign misconception of even symmetry

j 2f
31. The inverse F.T of X(f) = is ______
1  6 jf  8 2 f 2
1 4 t 1  1 
(A) e u t   e  2 t u t  (B) e  t  e  t / 2  u t 
2 2  2 
1   1 
(C)  e t  e t / 2  u t  (D) e t  e t / 2  u t 
2   2 

31. Ans: (B)


j 2f 1 1 1 1/ 2
Sol: Xf     = 
1  4 jf 1  2 jf  1  2 jf 1  4 jf 1  j 2f 1
 2 jf
2
1
Use e at u t  
a  j 2f

 1 
So, x t   e  t  e  t / 2  u t 
 2 
ACE Engineering Academy Hyderabad|Delhi|Bhopal|Pune|Bhubaneswar|Lucknow|Patna|Bengaluru|Chennai|Vijayawada|Vizag|Tirupati|Kukatpally|Kolkata|Ahmedabad
: 25 : EC

Distractor Logic:

Option (A): If you feel like by missing j2f in the numerator


1 1
1
X   2  2
4  j2  j 4  j 2  j
Option (B): Option (B) is correct
1
Option (C): If you adjust given form like X 
1  j1  j 
 2 
1
Option (D): If X  is not doing partial fraction expansion correctly
 j 
1  j1  
 2 

32. Consider (n, k) representing a LINEAR BLOCK CODE. Determine which of the following code
words represents a HAMMING CODE.
(A) (7, 4) (B) (6, 4) (C) (7, 3) (D) (9, 6)
32. Ans: (A)
Sol: For a code to be HAMMING CODE
2n–k = n + 1
(7, 4)
n=7
k=4
27 – 4 = 7 + 1
8=8
Hence option (A) is correct
None of the other options satisfies this criteria.

Destructor Logic:
(a) Option (A) is correct
(b) If you consider formula as 2n – k > n + 1; you will get (6, 4)
(c) If you consider formula as 2n – m = n + 1; you will get (7, 3)
(d) If you consider formula as 2n – k < n + 1; you will get (9, 6)

ACE Engineering Academy Hyderabad|Delhi|Bhopal|Pune|Bhubaneswar|Lucknow|Patna|Bengaluru|Chennai|Vijayawada|Vizag|Tirupati|Kukatpally|Kolkata|Ahmedabad


: 26 : PRE‐GATE_2019

ACE Engineering Academy Hyderabad|Delhi|Bhopal|Pune|Bhubaneswar|Lucknow|Patna|Bengaluru|Chennai|Vijayawada|Vizag|Tirupati|Kukatpally|Kolkata|Ahmedabad


: 27 : EC

d2y dy
33. The general solution of x 2
 5x
2
 9 y  0 is
dx dx
(A) (C1 + C2x)e3x (B) (C1 + C2 lnx)x3
3
(C) (C1 + C2x)x3 (D) (C1 + C2 lnx) e x
(Here, C1 & C2 are arbitrary constants)
33. Ans: (B)

d2y dy
Sol: Given x 2 2
 5x  9y  0
dx dx
d
Put z = logx or x = ez and D =
dz
The given equation becomes,
D(D – 1)y – 5Dy + 9y = 0
(D2 – 6D + 9)y = 0
The Auxiliary equation is D2 – 6D + 9 = 0
Roots are 3, 3
The general solution is y = (C1 + C2 z)e3z
 y = (C1 + C2 lnx)x3
Hence, option (B) is correct.

34. When a fair coin is tossed 200 times then mean and standard deviation are

(A) 100, 50 (B) 100, 50 (C) 50, 100 (D) 100 , 50


34. Ans: (B)
1 1
Sol: n = 200, p = ,q=
2 2
1
Mean = E(X) = np = 200  = 100
2
1 1
Variance = V(X) = npq = 200   = 50
2 2

Standard deviation = 50

ACE Engineering Academy Hyderabad|Delhi|Bhopal|Pune|Bhubaneswar|Lucknow|Patna|Bengaluru|Chennai|Vijayawada|Vizag|Tirupati|Kukatpally|Kolkata|Ahmedabad


: 28 : PRE‐GATE_2019

35. The Recursion relation to solve x = ex using Newton-Raphson method is ______.


(A) xn+1 = e x n (B) xn+1 = xn – e x n

(C) xn+1 =
1  x n e x n

(D) xn+1 =
1  x n e x n

1  e xn 1  e xn
35. Ans: (C)
Sol: Given equation x = ex

Let f(x) = x – ex

 f (x) = 1 – ex

By Newton-Raphson Formula

f x n 
xn+1 = x n 
f x n 

= xn 
x  e xn
n 
1  e xn

x n  x n e xn  x n  e xn
=
1  e xn

xn+1 =
1  x n e x n

1  e xn

Q. 36 – Q. 65 carry Two marks each.

36. The following two-port network described by Z-parameters matrix. Then the Thevenin’s equivalent
resistance across load RL in kilo ohms is ______________

I1 I2 5k

10k  1k  +
10k Z  V2 RL  15V
5mA V1 40k 8k 

ACE Engineering Academy Hyderabad|Delhi|Bhopal|Pune|Bhubaneswar|Lucknow|Patna|Bengaluru|Chennai|Vijayawada|Vizag|Tirupati|Kukatpally|Kolkata|Ahmedabad


: 29 : EC

36. Ans: 3.33 (Range 3.2 to 3.5)


Sol: For Rth (V S.C, IO.C)

I1 I2 5k

10k  1k 
O.C 10k V1 Z  V2 RL
40k 8k 

Req across output terminals of two-port network

5k
V Rth=Req||5k
R eq  2 then Req Rth
I2

For Req From input port & Z-parameters


V1 = 10kI1  (1)
Z-parameters
V1 = 10kI1  1kI2  (2)
V2 = 40kI1 + 8kI2  (3)

From 1 & 2 10kI1 = 10kI1  kI2


I2
I2 = 20I1  I1 
20
From 3
I 
V2  40K 2   8kI 2
 20 
V
V2 10kI 2  R eq  2 10k
I2

10
R th  R eq || 5k 10k || 5k  k  3.33k
3
ACE Engineering Academy Hyderabad|Delhi|Bhopal|Pune|Bhubaneswar|Lucknow|Patna|Bengaluru|Chennai|Vijayawada|Vizag|Tirupati|Kukatpally|Kolkata|Ahmedabad
: 30 : PRE‐GATE_2019

37. An input voltage VS(t) = 50 2 Cost volts with an internal impedance ZS = (1 + j 3.33) feeds a
load circuit as shown in figure below. The reactive power consumed by a load is _______ VAR

ZS
I
1 2
VS 
j2 –j2

Load

37. Ans: 66.67 (Range: 65 to 68)


Sol: The load impedance ZL = (1 + j2) || (2 – j2)

ZL 
1  j22  j2  2  4  j4  j2
3 3
 2 
Z L   2  j 
 3
Total impedance ZT = ZS + ZL = (1 + j 3.33) + (2 + j 0.67)
ZT = (3 + j4)
VS 50rms  500
I    10  53.14
Z T 3  j 4 553.14 

The Reactive power absorbed by the load circuit


QL = I2 XLoad
 2  200
Q L  10 2    VAR  66.67 VAR
3 3

38. The system and its unit step response are shown in figure a & b. The value of K/ is______
C(t)

+ K
R(s) C(s)
 S(1  S) 0.254
1
Figure (a)

0 t(sec)
3
Figure (b)
ACE Engineering Academy Hyderabad|Delhi|Bhopal|Pune|Bhubaneswar|Lucknow|Patna|Bengaluru|Chennai|Vijayawada|Vizag|Tirupati|Kukatpally|Kolkata|Ahmedabad
: 31 : EC

38. Ans: 1.3 (Range: 1.2 to 1.4)


C(s) K K/
Sol: CLTF  2  2
R (s) S   S  K S  S /   K / 
CE S2  S  K  0 ---(1)
 
Given mp = 0.254
1  2
0.254  e   /

 1.37    / 1   2 
  0.399  0.4
Given tp = 3 sec

3
n 1   2
n = 1.14 rad/sec
CE = S2 + 2ξnS + n2 = 0
CE = S2+20.41.14S+1.142 = 0
CE = S2+0.912S+1.3 = 0 ---(2)
Compare equation (1) and (2)
1 K
 0.912 & 1.3
 

39. P-type silicon can be doped in the range from 51014 cm3 to 1020 cm3 .The maximum possible
number of minority electrons in a neutral Ptype region is ______, if the device area is limited to
Ap = 1cm 1cm and the thickness of the Ptype region is limited to tp = 100m. Assume room
temperature and full acceptor ionization. [for silicon ni = 1.02  1010 cm3 at T = 300K]
39. Ans: 2080 (2078 to 2082)
Sol: The maximum concentration of minority carriers is obtained for the minimum doping level

n
n i 
2

1.02 1010 2

 0.20808  10 5 cm 3  2.08  1011 m 3


NA 5  1014
The maximum volume is
Volume = Ap  tp = 0.01m  0.01m  100  106m = 108 m3
The maximum number of minority electrons in this volume is
N = n  volume = 2.08  1011m3  108m3 = 2.08  103 = 2080
ACE Engineering Academy Hyderabad|Delhi|Bhopal|Pune|Bhubaneswar|Lucknow|Patna|Bengaluru|Chennai|Vijayawada|Vizag|Tirupati|Kukatpally|Kolkata|Ahmedabad
: 32 : PRE‐GATE_2019

40. Having lost his 2.4V cell phone charger, an electronics engineering student tries several stores but
does not find adaptors with outputs less than 3V. He then decided to put his knowledge of
electronics to work and constructs the circuit shown in fig. Where three identical diodes in forward
bias produce a total voltage of Vout = 3 VD  2.4V and resistor R sustains the remaining 600mV.
Neglect the current drawn by the cell phone. The reverse saturation current Is is ______(in fA), so
 kT 
that Vout = 2.4V. Assume   1,  26mV at T  300k 
 q 

I
+
600mv R=100
+ 
+
Vad=3V Adaptor
 Cellphone
Vout

40. Ans: 0.26 (0.25 to 0.27)


Sol: With Vout = 2.4V, the current flowing through R is equal to

Vad  Vout 3V  2.4V


I   6mA
R 100

We note that each diode carrier I and hence

I  Is e VD / VT

800 mv
6mA  I s . e 26 mv

6mA
 Is  800 mv
e 26 mv

= 2.602  1016 A
= 0.26  1015 A
= 0.26  1015 A
ACE Engineering Academy Hyderabad|Delhi|Bhopal|Pune|Bhubaneswar|Lucknow|Patna|Bengaluru|Chennai|Vijayawada|Vizag|Tirupati|Kukatpally|Kolkata|Ahmedabad
: 33 : EC

41. If gm is the transconductance, r0 is the output resistance and r is the base to emitter dynamic
V 
resistance then calculate the approximate small signal voltage gain  0  is _______.
 VS 

+V

Q1

V0

Q2
+
VS

Q3

–V

41. Ans: –0.5 (No range)


Sol:

1 1
|| r 
gm gm
V0

+
VS
– 1
gm

1

V0 gm
  0.5
VS 1 1

gm gm
V0  1
   0 .5
VS 2

ACE Engineering Academy Hyderabad|Delhi|Bhopal|Pune|Bhubaneswar|Lucknow|Patna|Bengaluru|Chennai|Vijayawada|Vizag|Tirupati|Kukatpally|Kolkata|Ahmedabad


: 34 : PRE‐GATE_2019

ACE Engineering Academy Hyderabad|Delhi|Bhopal|Pune|Bhubaneswar|Lucknow|Patna|Bengaluru|Chennai|Vijayawada|Vizag|Tirupati|Kukatpally|Kolkata|Ahmedabad


: 35 : EC

42. A Helmholtz coil consists of two circular coaxial coils each of single turn and radius 2cm,
separated by a distance of d = 0.02m. If the two coils carry equal currents I = 5Amp in the same
direction shown in figure and placed in vacuum, then the magnitude of magnetic flux density
(in micro Tesla) at a point ‘P’ midways between the coils is ______

I I
P z
2 cm
d

42. Ans: 224.7 (Range: 222 to 226)


Sol: Magnetic flux density on the axis of circular current loop at z = h is given by
 0 IR 2
B1  â z
2(R 2  h 2 ) 3 / 2

As the two loops are carrying current ‘I’ in the same direction and hence

2 0 IR 2
B  B1  B 2  â z
2( R 2  h 2 ) 3 / 2
Magnitude of magnetic flux density is

2 0 IR 2
B
2( R 2  h 2 ) 3 / 2
Given: radius R = 2cm

Spacing d = 0.02 m (or) 2cm

d R
At midpoints, h  
2 2

2 0 IR 2 8 0 IR 2 8 0 I 8  4  10 7  5
B 3/ 2
   = 22.4710–5 = 224.710–6
 R  2
5 5R 3
5 5R 5 5  2  10 2

2 R 2  
 4 

 B  224.7T

ACE Engineering Academy Hyderabad|Delhi|Bhopal|Pune|Bhubaneswar|Lucknow|Patna|Bengaluru|Chennai|Vijayawada|Vizag|Tirupati|Kukatpally|Kolkata|Ahmedabad


: 36 : PRE‐GATE_2019

 t  0.01 
43. A system has an impulse response h(t) = 10 rect   . The null-to-null B.W. is _____ Hz
 0.02 
43. Ans: 50 (no range)
 T 
Sol: A rect(t/T)  ATSa  
 2 
 t  0.01 
Given h(t) = 10 rect  
 0.02 
    j0.01
H() = 0.2 Sa(0.01) e–j(0.01) = 0.2 Sa  e
 100 

zero crossing frequencies are   n
100
  100n

Null to null bandwidth = 100 rad/sec = 50Hz

44. A system with input x(n) and output y(n) are related as y(n) – 0.2y(n – 1) = x(n) – 5x(n – 1).
The magnitude of the d.c & high frequency gain of this filter are  and  respectively.
Then the value of  +  is _______
44. Ans: 10 (no range)

Sol: y(n) – 0.2y(n–1) = x(n) – 5x(n–1)

Apply z-transform
Y(z) – 0.2 z–1 Y(z) = X(z) – 5z–1 X(z)

Yz  1  5z 1 1  5e  j
Transfer Function is H(z) =   H e j
 
Xz  1  0.2z 1 1  0.2e  j

1 5
D.C. gain = He j0    5= 
1  0.2

1 5
H.F. gain = He j   5 =
1  0.2

 +  = 10
ACE Engineering Academy Hyderabad|Delhi|Bhopal|Pune|Bhubaneswar|Lucknow|Patna|Bengaluru|Chennai|Vijayawada|Vizag|Tirupati|Kukatpally|Kolkata|Ahmedabad
: 37 : EC

2
45. The autocorrelation of a random process X(t) is Rxx() = 3 + 2 e 4  .

The fraction of the power lies in the frequency band – 2    2 is _____________


(Assume Q (0.5) = 0.3)
45. Ans: 0.76 (Range: 0.75 to 0.77)
2
Sol: Rxx() = 3 + 2 e 4   Px = RXX(0) = 3 + 2 = 5W
2
 
 Sxx() = 3(2 ()) + 2 e 4 ( 4 )
4


Power within  2  0  2 is 
2
1 2 1 1 2   28
 xx
S ( ) d   3( 2  )  2  e d
2  2 2 2  2 4

Let  x  d  2 2dx
2 2
1 1
2 2
2 2   x2 2 2  x2
 3  e ( 2 2 ) dx  3   e dx
2  1 4 2  1
2 2

  1 
 3  21  2Q 
  2 
 3  2[1  0.6]
= 3.8W
3.8
Fraction of power within  2    2 is,  0.76
5

46. Consider a signal detector with an input


r=A+n
where + A and – A occur with equal probability and the noise variable ‘n’ is characterized by the
laplacian pdf as shown in the figure below:
1 |n|/ 2
f(x) = e
2

0 n

ACE Engineering Academy Hyderabad|Delhi|Bhopal|Pune|Bhubaneswar|Lucknow|Patna|Bengaluru|Chennai|Vijayawada|Vizag|Tirupati|Kukatpally|Kolkata|Ahmedabad


: 38 : PRE‐GATE_2019

If the threshold voltage Vth is considered as zero i.e

Vth  0  decoded as A

Vth < 0  decoded as –A

Then the value of A(in volts) is __________ such that the Pe = e–1

46. Ans: 1.414 (Range: 1.4 to 1.5)

Sol: r =  A + n  0  decoded as A

r =  A + n < 0  decoded as –A

 A + n < 0  A will be decoded as –A

 n < –A

– A + n  0  –A will be decoded as A

nA

A 
 Pe  P(A)  f ( x )dx  P(A)  f ( x )dx
 A

1 1 n/ 2
A
1 1 n / 2 

Pe   e dn   e dn
2  2 2A 2

Area’s under both these two functions are same as the given pdf as symmetrical w.r.t origin

1   1 n / 2 
 Pe  2  e dn 
2  A 2 

Pe 
1
e n / 2


A  2
2

Pe = e  A / 2

But given Pe should be e–1

 eA / 2
= e–1

Hence A = 2 = 1.414 Volts


ACE Engineering Academy Hyderabad|Delhi|Bhopal|Pune|Bhubaneswar|Lucknow|Patna|Bengaluru|Chennai|Vijayawada|Vizag|Tirupati|Kukatpally|Kolkata|Ahmedabad
: 39 : EC

47. On the average 15 cars pass a certain point on a road per hour. What is the probability that exactly
four cars pass through the point in a 12 minute period?
47. Ans: 0.168 (Range: 0.16 to 0.17)
Sol: Let we use Poisson distribution.
15
Average number of cars passing through the point in a 12 minute period = 3
 60 
 
 12 
=3
By Poisson distribution,
e   .k
P(X = k) =
k!
e 3 .34
 The Required probability = P(X = 4) = = 0.168
4!

TWO MARK (MCQ)


48. Consider the following circuit as shown in figure. Switch is opened for a long time and it is closed
at t = 2 sec. Determine the value of iL at t = 3 sec

8
S 4
12A t=2s 4H
iL

4 4 4 4
(A) A (B) A (C) A ( D) A
e2 e6 e10 e3
48. Ans: (A)
Sol: For t < 2, ‘S’ is opened
t = 2–  (steady state) LS.C for initial value of ‘L’

8
12A 4
iL(2)

By Current Division Rule


12  4
i L 2     4A  i L 2    I 0
48
ACE Engineering Academy Hyderabad|Delhi|Bhopal|Pune|Bhubaneswar|Lucknow|Patna|Bengaluru|Chennai|Vijayawada|Vizag|Tirupati|Kukatpally|Kolkata|Ahmedabad
: 40 : PRE‐GATE_2019

For t > 2, ‘S’ is closed

8
12A
4H
iL(2)

It is RL source free circuit


L 4 1
  
R 8 2
i L 2   I o e  t 
t 2 
1

i L t   4e 2

i L t   4e  2  t  2  A
At t  3 Sec
iL(t) = 4 e2(32)
4
i L t   Amps
e2

Distractor Logic:
(a) Option (A) is correct
(b) If iL(t) = 4 e2t
At t = 3 sec iL(t) = 4 e2(3)
4
i L t   A
e6
(c) If iL(t) = 4 e2(t+2)
At t = 3 sec iL(t) = 4 e2(3+2) = 4 e10
4
i L t   A
e10
(d) If iL(t) = 4 et
At t = 3 sec iL(t) = 4 e(3)
iL(t) = 4e3
4
i L (t)  A
e3

ACE Engineering Academy Hyderabad|Delhi|Bhopal|Pune|Bhubaneswar|Lucknow|Patna|Bengaluru|Chennai|Vijayawada|Vizag|Tirupati|Kukatpally|Kolkata|Ahmedabad


: 41 : EC

49. The asymptotic magnitude plot of a minimum phase system is shown in figure.
The approximate phase margin of the system is

M(dB)

36 20 dB/dec
0 dB/dec
20 dB/dec
6
100
0 2 
1 20 40 dB/dec

(A) 45o (B) 50o (C) 55o (D) 60o


49. Ans: (D)
Sol: Select line from 2 rise to 100 rad/sec.
 M 2  M1   06 
Slope     20   
 log  2  log 1   log 100  log 2 
6
 log 100  log  2   0.3
 20
log2 = 0.32

log2 = 1.7

2 = 50.11  50 rad/sec

K 1  S / 20 
 TF G (s)H(s) 
S1  S / 501  S / 100

gc = 100rad/sec [from figure]

  gc      
PM  180 0  G  jH j |gc  180 0   90 0  tan 1    tan 1  gc   tan 1  gc 
  20   50   100 

  100  1  100  1  100  


PM  180 0   90 0  tan 1    tan    tan    60
0

  20   50   100 

ACE Engineering Academy Hyderabad|Delhi|Bhopal|Pune|Bhubaneswar|Lucknow|Patna|Bengaluru|Chennai|Vijayawada|Vizag|Tirupati|Kukatpally|Kolkata|Ahmedabad


: 42 : PRE‐GATE_2019

Distractor Logic:
(a) If 2 = 20 rad/sec, then PM = 450
(b) If 2 = 30 rad/sec, then PM = 500
(c) If 2 = 40 rad/sec, then PM = 550
(d) Correct ans: if 2 = 50 rad/sec, then PM = 600

50. A linear time invariant system is described by the state equation

X   1 2  0 
X    U . The state transition matrix is
 1 3
 1 
e cost   sin t 
2 t
2e 2 t sin t  
(A)   2 t 
e sin t  e  2 t cost   sin t 

 e 2 t cost   sin t  2e 2 t sin t  


(B)   2 t 
 e sin t  e  2 t cost   sin t 

e 2 t cost   sin t  2e 2 t sin t  


(C)   2 t 
 e sin t  e  2 t cost   sin t 

e 2 t cost   5 sin t  2e 2 t sin t  


(D)   2 t 
e sin t  e cost   3 sin t 
2 t

50. Ans: (B)


S  1 2 
Sol: SI  A  
 1 S  3
 AdjSI  A 

STM ( t )  L1 SI  A 
1
 L 1
 | SI  A | 
 
 S3 2   S  2  1 2 
2 
 S  2 2  1 S  2  1 1  S  22  12
2  S  2  1 
2

t   L1  L
 1 S 1   1 S  2  1 
 S  2 2  1

2 
S  2  1 
 S  2  1
2 2
S  22  12 

e 2 t cost   sin t  2e 2 t sin t  


  2 t 
 e sin t  e  2 t cost   sin t 

ACE Engineering Academy Hyderabad|Delhi|Bhopal|Pune|Bhubaneswar|Lucknow|Patna|Bengaluru|Chennai|Vijayawada|Vizag|Tirupati|Kukatpally|Kolkata|Ahmedabad


: 43 : EC

Distractor Logic:
 S 1 2 
 S  2 2  1 S  2  1
2

(a) If AdjSI  A    , then (t) becomes option (A)


 1 S3 
 S  2 2  1
 S  22  1
(b) Option (B) is correct

 S3 2 
 S  2 2  1 S  2  1
2

(C) If AdjSI  A   , then (t) becomes option (C)


 1 S 1 
 S  2 2  1
 S  22  1

(D) If [SIA] is write wrong, then (t) becomes option (D)

51. A diode whose V-I characteristics is shown in figure (b) is connected as in figure (a). Assuming
emission coefficient as 2, calculate reverse saturation current. (VT = 26mV)

ID(mA)

9.01
R
Vin + Q
– ID
+
VD

Figure (a) 0 0.5 0.6 3V VD(Volts)
Figure (b)

(A) 0.685 pA (B) 0.714 pA

(C) 73.2 nA (D) 70.3 nA

51. Ans: (D)

ACE Engineering Academy Hyderabad|Delhi|Bhopal|Pune|Bhubaneswar|Lucknow|Patna|Bengaluru|Chennai|Vijayawada|Vizag|Tirupati|Kukatpally|Kolkata|Ahmedabad


: 44 : PRE‐GATE_2019

Sol: Vin = IDR + VD ---- (1)


Substitute, ID = 0 and VD = 3V in equation (1)  Vin = 3V

Substitute, VD = 0 and ID = 9.01mA in equation (1)

Vin 3
R   332.96
ID 9.01  10 3

 R  333 

Vin  VDQ 3  0.6


I DQ    7.21mA,  = 2 (Emission coefficient)
R 333

VDQ / VT
IDQ = I 0 (e  1)

I DQ 7.21  103
 Reverse saturation current, I o    70.3nA
 VDQ   200..6026 
 e 2 VT  1 e  1
  
   

Destructor Logic:

(a) If  = 1 then we get I0 = 0.685pA

(b) If VDQ = 0.5V &  = 1 then I0 = 0.714pA

(c) If VDQ = 0.5V &  = 2 then I0 = 73.2nA

(d) Option (D) is correct

52. In the circuit shown BJT is in active region with RB = 100k & VBC = 0V. It is observed that if RB

decreases then transistor moves into saturation. If RB changes to 10% of its original value VBC

becomes 0.6V. Calculate IC.

Assume  = 100, VBE (Active) = 0.7V, VBE (sat) = 0.8V

ACE Engineering Academy Hyderabad|Delhi|Bhopal|Pune|Bhubaneswar|Lucknow|Patna|Bengaluru|Chennai|Vijayawada|Vizag|Tirupati|Kukatpally|Kolkata|Ahmedabad


: 45 : EC

+5V +5V

RB 1k

1k

(A) 2.486 mA (B) 2.314 mA (C) 2.68 mA (D) 2.134 mA

52. Ans: (B)


Sol:
5V 5V

RB 10k 1k RC
IB IC
VBC– C VC
+ N +
VB B P VCE
+ –
N
VBE E VE
– IE
1k R
E

RB = 10% of original value

= 10% of 100k

= 10k

RB decreased  Saturation region

Apply KVL around transistor

VCE (sat) + VBC(sat) = VBE(sat)  VCE(sat) = 0.8 – 0.6 = 0.2V

Apply KCL: IE = IC + IB

ACE Engineering Academy Hyderabad|Delhi|Bhopal|Pune|Bhubaneswar|Lucknow|Patna|Bengaluru|Chennai|Vijayawada|Vizag|Tirupati|Kukatpally|Kolkata|Ahmedabad


: 46 : PRE‐GATE_2019

VE 5  VC 5  VB
 
1k 1k 10k

VE 5  [VCE (sat )  VE ] 5  [VBE ( sat )  VE ]


  
1k 1k 10k

 VE = 2.486 V

 VC = VCE (sat) + VE = 2.686V

5  VC
 IC =  2.314mA
1k

Destructor Logic:

(a) Is Wrong
VE
IE   2.486mA
RE
If IB neglected then IC = IE = 2.486mA

(b) Option (B) is correct

(c) Is Wrong
VC
IE   2.686mA
RC
5  VC V
IC is not C
RC RC

(d) Is Wrong
5  VC
IC = = 2.314mA
RC
Data Inverted looks like 2.134mA

ACE Engineering Academy Hyderabad|Delhi|Bhopal|Pune|Bhubaneswar|Lucknow|Patna|Bengaluru|Chennai|Vijayawada|Vizag|Tirupati|Kukatpally|Kolkata|Ahmedabad


: 47 : EC

ACE Engineering Academy Hyderabad|Delhi|Bhopal|Pune|Bhubaneswar|Lucknow|Patna|Bengaluru|Chennai|Vijayawada|Vizag|Tirupati|Kukatpally|Kolkata|Ahmedabad


: 48 : PRE‐GATE_2019

53. Calculate VBE in the following circuit where Q2 is formed as the parallel combination of “m”
units each identical to Q1 [Vt is the thermal voltage]

+VDD

nI0 I0

+ V –
BE

Q1 Q2

‘m’ units

n
(A) Vt ln [n m] (B) Vt ln  
m

m  1 
(C) Vt ln   (D) Vt ln  
n  nm 

53. Ans: (A)

I  I   nI  I / m
Sol: VBE = VB – VE = Vt ln  C1   Vt ln  C 2  = Vt ln  0   Vt ln  o 
 IS   IS   IS   IS 

= Vt ln [n m]

Distractor Logic:
Option A: Right answer

Option B: If current division is not applied in Q2

Option C: If current division is applied in Q1

Option D: If  VEB is considered

ACE Engineering Academy Hyderabad|Delhi|Bhopal|Pune|Bhubaneswar|Lucknow|Patna|Bengaluru|Chennai|Vijayawada|Vizag|Tirupati|Kukatpally|Kolkata|Ahmedabad


: 49 : EC

54. A combinational circuit has 4 inputs X3, X2, X1, X0 (LSB) and 4 outputs Y3, Y2, Y1, Y0 (LSB) and
its implementation is as shown below. Find the function of the circuit.

X3 X2 X1 X0

Y3

Y2

Y1

Y0

(A) Binary to Gray code converter (B) Gray to Binary code converter
(C) 2’s complement of Binary input (D) BCD to Ex-3 code converter
54. Ans: (C)
Sol: From given combinational circuit
 Y3 = (X2 + X1 + X0)  X3
Y2 = (X1 + X0)  X2
Y1 = X0  X1
Y0 = 0  X0 = X0
It is 2’s complement of binary input X3 X2 X1 X0

Distractor logic:
(A): If output expressions are taken as Y3 = X3
Y2 = X2  X3
Y1 = X1  X2
Y0 = X0  X1
Then it acts as “Binary to Gray” code converter
(B): If output expressions are taken as Y3 = X3
Y2 = X3  X2
Y1 = Y2  X1
Y0 = Y1  X0
Then it acts as “Gray to Binary” code converter
(C): Option (C) is correct
(D): Possibility to pick it as “BCD to Ex-3 code converter”.

ACE Engineering Academy Hyderabad|Delhi|Bhopal|Pune|Bhubaneswar|Lucknow|Patna|Bengaluru|Chennai|Vijayawada|Vizag|Tirupati|Kukatpally|Kolkata|Ahmedabad


: 50 : PRE‐GATE_2019

55. A 4-Bit Digital comparator with X<Y and X=Y outputs is built using two 4-Bit Binary subtractors
as shown in the following figure, where P and Q indicates the End Around Carry (EAC) value
X Y

4 4
2’S Complement 2’S Complement
circuit circuit
4 4 4 4
P 4-Bit Q 4-Bit
X<Y Logic C5 Binary C5 Binary
ckt 1 1 Adder Adder
Logic 4 4
X=Y
ckt 2

Which of the following is suitable for implementation of logic circuit 1 and 2

(A) (B)
P P
1 1 1 1

2 Q 2 Q

(C) 1 P (D) 1 P
1 1

2 Q 2 Q

55. Ans: (C)


Sol: If EAC = 0 i.e., P = 0 then X < Y output is P  1 = 0  1 = 1
X = Y only if P = 1, Q = 1 then X = Y output is P.Q

Distractor logic
(A) If P is mistaken for carry and equality occurs if either P or Q is 1.
(B) If outputs P, Q are taken as carries after subtraction.
(C) Option (C) is correct
(D) If output P is mistaken for carry instead of EAC.
ACE Engineering Academy Hyderabad|Delhi|Bhopal|Pune|Bhubaneswar|Lucknow|Patna|Bengaluru|Chennai|Vijayawada|Vizag|Tirupati|Kukatpally|Kolkata|Ahmedabad
: 51 : EC

56. An XY latch is constructed using OR, AND and inverter gates as shown below.
The values of P and Q must be complement to each other.
Find the output waveform, for the following inputs

X P X
t1
Y
Q
Y t2 t3

(A) (B)

t1 t2 t3 t1 t2 t3

(C) (D)

t1 t2 t3 t1 t 2 t3

56. Ans: (D)


Sol: XY latch function table is
X Y P Q
0 0 0 1
0 1 No Change
1 0 Not Allowed
1 1 1 0

Sketch the waveform at 'P' based on X, Y inputs according to the above table.

Destruction logic:
(a) If 01 and 00 input response is taken as 0 and 1 respectively
(b) If 11, 01, 00 Input responses are taken as 0, 1, 0 respectively
(c) If 11, 01, 00 input response is taken as 0, 0, 1 respectively
(d) Option (D) is correct
ACE Engineering Academy Hyderabad|Delhi|Bhopal|Pune|Bhubaneswar|Lucknow|Patna|Bengaluru|Chennai|Vijayawada|Vizag|Tirupati|Kukatpally|Kolkata|Ahmedabad
: 52 : PRE‐GATE_2019


57. If an end fire uniform linear array with the spacing between the elements has directivity of
4
20dB, then the number of elements required and beam width between first nulls respectively will
be
(A) 101 & 32.400 (B) 201 & 11.460
(C) 101 & 16.200 (D) 201 & 2.300
57. Ans: (A)
Sol: Given: 10logD = 20

D = 102 = 100

4
Directivity D 

4n  1d
D


4n  1
100  4

 Number of elements required, n = 101

Beam width between first Nulls:

2 2
BWFN or  FNBW  2 2
 n  1d

2
2

100 
4

8
2
200

 180  0
FNBW  2 0.08     32.40
  

ACE Engineering Academy Hyderabad|Delhi|Bhopal|Pune|Bhubaneswar|Lucknow|Patna|Bengaluru|Chennai|Vijayawada|Vizag|Tirupati|Kukatpally|Kolkata|Ahmedabad


: 53 : EC

Destruction logic:

(a) Option (A) is correct

2
(b) D


n  1
100  4

n = 201

2 2 8 180
FNBW     = 11.460
n  1d 200   200 
4

4
(c) D

n = 101

2 2 8 180
FNBW     16.200
n  1d 100   100 
4
2
(d) D


2n  1
100  4

 n = 201
2 2 8
FNBW   
n  1d 200   200
4
 FNBW = 2.300

ACE Engineering Academy Hyderabad|Delhi|Bhopal|Pune|Bhubaneswar|Lucknow|Patna|Bengaluru|Chennai|Vijayawada|Vizag|Tirupati|Kukatpally|Kolkata|Ahmedabad


: 54 : PRE‐GATE_2019

58. A rectangular waveguide of 6 cm ×4 cm, filled with a dielectric has refractive index of 1.5.
Over what range of frequencies (in GHz) will the guide allows only dominant mode and rejects
TE01 mode ?
(A) 2.5 < f < 3.75 (B) 5 < f < 7.5 (C) 3.3 < f < 5 (D) 1.67 < f < 2.5
58. Ans: (D)
Sol: Given: dimensions a = 6 cm,
b = 4 cm
refractive index n = 1.5
To allow dominant mode: f > fc (TE10)
to reject TE01 mode : f < fc (TE01)
c 3  1010
f c TE 10     1.67 GHz
2na 2  1.5  6
c 3  1010
f c TE 01     2.5 GHz
2nb 2  1.5  4
therefore the range of frequencies, over which, the guide allows only dominant mode (TE10) and
rejects TE01 mode is
1.67 GHz < f < 2.5 GHz

Distractor Logic:
c 3  1010
(a) f c TE 10     2.5 GHz
2a 26

c 3  1010
f c TE 01     3.75 GHz
2b 2  4

 Frequency range: 2.5 GHz < f < 3.75 GHz, which is wrong answer

c 3  1010
(b) f c TE 10     5GHz
a 6

c 3  1010
f c (TE 01 )    7.5 GHz
b 4

Frequency range: 5 GHz < f < 7.5 GHz, which is wrong answer.

ACE Engineering Academy Hyderabad|Delhi|Bhopal|Pune|Bhubaneswar|Lucknow|Patna|Bengaluru|Chennai|Vijayawada|Vizag|Tirupati|Kukatpally|Kolkata|Ahmedabad


: 55 : EC

c 3  1010
(c) f c TE 10     3.3 GHz
na 1.5  6
c 3 1010
f c TE 01     5 GHz
nb 1.5  4
 Frequency range: 3.3 GHz < f < 5 GHz, which is wrong answer
(d) Option (D) is correct

59. An analog signal x(t) = cos(0.25t) is passed through the following system.

x(t) y(t)
Sampler H(ej) Ideal L.P.F

The sampler is ideal and operates at fs = 1Hz. The cut-off frequency of the ideal low pass filter is
fc = 0.5 Hz. The frequency response of the digital filter is H(ej) = rect(/)e–j/4. The relation
between y(t) and x(t) is _______
(A) y(t) = x(t) (B) y(t) = x(t – 0.25)
(C) y(t) = x(t – 0.5) (D) y(t) = x(t + 0.5)
59. Ans: (B)
Sol: Given x(t) = cos(t/4) with fs = 1Hz
 n 
Resultant discrete time signal is x(n) = cos  
 4 
o = /4
H(ej) = rect(/) e–j/4

 
 
H e j  e
j
4

2

2

 n  
Thus, y(n) = cos   
 4 16 

 t    
 y(t) = cos    = cos  t  0.25 = x(t – 0.25)
 4 16  4 

ACE Engineering Academy Hyderabad|Delhi|Bhopal|Pune|Bhubaneswar|Lucknow|Patna|Bengaluru|Chennai|Vijayawada|Vizag|Tirupati|Kukatpally|Kolkata|Ahmedabad


: 56 : PRE‐GATE_2019

Distractor logic:
Option (A): If we feel He j   1 acting like all pass filter, we may think y(t) = x(t)

Option (B): Option (B) is correct



Option (C): If we are not taking common multiplier in the output term correctly, it may be
4
treated as option (C).
Option (D): If we feel time advance, may be choosing option (D)

60. A signal x(t) is band–limited to 40Hz & modulated by a 320Hz carrier to generate modulated
signal y(t). The minimum sampling rate for y(t) to prevent aliasing is ______
(A) > 80 Hz (B) 160 Hz
(C) 180 < fs < 186.67 Hz (D) 360 < fs < 373 Hz

60. Ans: (C)


Sol: 280Hz = fL
40Hz 32040
Modulated at 320Hz 360Hz = fH

X(f)

f
–360 –280 280 360

 f   360 
N  int  H   int  4
 fH  fL   80 
2f 2f L
we require H  f S  ; m = 1,2,----N
m m 1
= 1, 2, 3, 4

2f H 2f
For minimum fs, m = 4   fS  L
4 3
180  fs  186.67 Hz

ACE Engineering Academy Hyderabad|Delhi|Bhopal|Pune|Bhubaneswar|Lucknow|Patna|Bengaluru|Chennai|Vijayawada|Vizag|Tirupati|Kukatpally|Kolkata|Ahmedabad


: 57 : EC

Distractor logic:
Option (A): Assuming y(t) as band limited signal taking fs > 80 Hz
320
Option (B): As carrier frequency is 320 Hz, you may feel  160 Hz
2
Option (C): Option (C) is correct
Option (D): If you feel least value of m = 2, you may choose option (D)

61. The system shown in the figure below is used to generate a standard AM signal

Non-linear y(t)
m(t) x(t) Linear s(t) AM signal
+ Memory less Filter
System

c(t) = cos(2fct)

The non-linear device has input characteristics given by y(t) = ax(t) + bx2(t) and m(t) is a band
limited signal having peak amplitude ‘Am’ and maximum frequency ‘fm’ Hz [Am = max|m(t)|].
The modulation index ‘’ is
2bA m bA m
(A) (B) 2bA m (C) (D) bA m
a a
61. Ans: (A)
Sol: y( t )  a[m( t )  cos 2f c t ]  b[m( t )  cos 2f c t ]2

b b
y( t )  am( t )  a cos 2f c t  bm 2 ( t )   cos 4f c t  2bm( t ) cos 2f c t
2 2
y(t)  contains (a) signal component’s m(t) and m2(t)
(b) Carrier components at fc, 2fc
(c) DC component
(d) USB and LSB components
Since s(t) of a standard AM is required we have to consider carrier components (fc),
USB [fc + fm] and LSB [fc – fm] components.
 2bm( t ) 
 s(t)AM signal = acos2fct + 2bm(t) cos2fct = a 1  cos 2f c t
 a 
2b 2bA m
 [Modulation Index] = | Max (m( t )) | =
a a
ACE Engineering Academy Hyderabad|Delhi|Bhopal|Pune|Bhubaneswar|Lucknow|Patna|Bengaluru|Chennai|Vijayawada|Vizag|Tirupati|Kukatpally|Kolkata|Ahmedabad
: 58 : PRE‐GATE_2019

Destructor logic:
(A) Option (A) is correct
(B) If student by mistake considers
s(t) = cos2fct + 2bm(t)cos2fct
= [1 + 2bm(t)] cos2fct
   2b | Max (m( t )) | 2bA m
(C) If s(t)AM = acos2fct + bm(t)cos2fct
 b 
= a 1  m( t ) cos2fct
 a 
bA m
 
a
(D) If s(t)AM = cos2fct + bm(t)cos2fct
= a 1  bm( t ) cos(2fct)
   bA m
62. The values of constants a and b for which the vector

     
   
F  x 2  y  a  b  z i  a  b x  y 2  z j  2x  y  z 2 k is irrotational, then which of the

following is correct ?

3 1 1 3
(A) a  ,b  (B) a  , b 
2 2 2 2

1 3 3 1
(C) a  ,b  (D) a  , b 
2 2 2 2

62. Ans: (D)

 
Sol: curl F    F = 0

  
i j k
  
0
x y z
x  y  a  b z
2
a  b x  y 2  z 2 x  y  z 2

ACE Engineering Academy Hyderabad|Delhi|Bhopal|Pune|Bhubaneswar|Lucknow|Patna|Bengaluru|Chennai|Vijayawada|Vizag|Tirupati|Kukatpally|Kolkata|Ahmedabad


: 59 : EC

  
 i  1  1  j 2  a  b   k a  b   1  0

     
 0 i  j a  b   2  k a  b   1  0 i  0 j  0 k
a – b – 2 = 0; a + b – 1 = 0
a–b=2
a+b=1
3 1
on solving these two equations, we get a  , b 
2 2
 Option (D) is correct.

63. For an nn matrix consisting of all ones, which of the following is true?

(A) the distinct eigen values are 0 and 1

(B) the distinct eigen values are 1 and n

(C) the distinct eigen values are 1, 2, ........., n

(D) the distinct eigen values are 0 and n


63. Ans: (D)
 1 1 ........ 1
 1 1 ........ 1 
Sol: Let A  
... .... ........ ...
 
 1 1 ......... 1  n n

|A –I| = 0

1   1 ........ 1 
 1 1   ........ 1 
 0
 ... .... ........ ... 
 
 1 1 ......... 1    n n

C1  C1 + C2 + .................... + Cn

ACE Engineering Academy Hyderabad|Delhi|Bhopal|Pune|Bhubaneswar|Lucknow|Patna|Bengaluru|Chennai|Vijayawada|Vizag|Tirupati|Kukatpally|Kolkata|Ahmedabad


: 60 : PRE‐GATE_2019

n 1 ........ 1
n   1   ........ 1
0
... .... ........ ...
n 1 ......... 1  

R2 – R1, R3 – R1, ........., Rn – R1

n   1 ........ 1
0   ........ 0
0
... .... ........ ...
0 0 .........  

  = n, 0, 0, .........., 0 [(n – 1) zeros]


 The distinct eigen values are 0 and n


 y  y 2  cot x , y   1 , is
dy
64. The solution of the initial value problem
dx 2
(A) y cosx – (1 – y) sinx = 0 (B) y = sinx
(C) y = sinx + cosx (D) 2y – (1 + y) sinx = 0

64. Ans: (D)

 y  y 2  cot x
dy
Sol: Given
dx
dy
 cot x dx
y  y2
dy
 cot x dx
y1  y 

1 1 
   dy  cot x dx
 y 1 y 
Integrating both sides, we get
n y – n (1 + y) = n sinx+ n C
ACE Engineering Academy Hyderabad|Delhi|Bhopal|Pune|Bhubaneswar|Lucknow|Patna|Bengaluru|Chennai|Vijayawada|Vizag|Tirupati|Kukatpally|Kolkata|Ahmedabad
: 61 : EC

 y 
n    n C sin x 
1 y 
y
 C sin x
1 y
 y = C(1 + y) sinx .............. (1) is the general solution.
 
y   1 , (1)  1 = C(1 + 1) sin  
2 2
 1 = 2C
1
C=
2
1
y= 1  y sin x
2
2y = (1 + y) sinx
2y – (1 + y)sinx = 0
Hence, option (D) is correct.

 z2 
e 2 
65. The value of the integral   3  dz , where C = {z: |z| = 1}, is equal to
C  z 
 

(A) 0 (B) 0.5 i (C) i (D) 2i

65. Ans: (C)

  z2 
2

z2
   
e 2
1   z2   2  
Sol: = 3 1      ...........
z3 z   2 2! 
 
 
1   z2  z4 
= 1      ...........
z3   2 8 
1 1 z
=    ........
z 3 2z 8

ACE Engineering Academy Hyderabad|Delhi|Bhopal|Pune|Bhubaneswar|Lucknow|Patna|Bengaluru|Chennai|Vijayawada|Vizag|Tirupati|Kukatpally|Kolkata|Ahmedabad


: 62 : PRE‐GATE_2019

1 1
At z = 0, residue of f(z) = coefficient of in the above expansion =
z 2
 By Cauchy residue theorem, we have
 z 
2

e2  1
  3  dz = 2i    = i
C z  2
 

ACE Engineering Academy Hyderabad|Delhi|Bhopal|Pune|Bhubaneswar|Lucknow|Patna|Bengaluru|Chennai|Vijayawada|Vizag|Tirupati|Kukatpally|Kolkata|Ahmedabad

You might also like